Autor Tema: Convergencias Lp y casi segura

0 Usuarios y 1 Visitante están viendo este tema.

09 Septiembre, 2023, 02:54 pm
Leído 151 veces

girsanov

  • $$\Large \color{#5372a0}\pi\,\pi$$
  • Mensajes: 80
  • País: es
  • Karma: +0/-0
  • Sexo: Masculino
Buenas, estoy un poco atascado con un problema, a ver si alguien sabe cómo resolverlo. El enunciado es el siguiente:

Sea un espacio de medida \( (\Omega, \mathcal{A}, \mu) \), sea \( 1 \leq p \leq \infty \) y sea \( \{f_n\}_{n=1}^{\infty} \) una sucesión de funciones, todas pertenecientes a \( L^p(\Omega, \mathcal{A}, \mu) \) así como una función \( f \in L^p(\Omega, \mathcal{A}, \mu) \) de modo que
\( \displaystyle \sum_{n=1}^{\infty} \lVert f_n - f \rVert_{p} <\infty. \)

Se pide demostrar que la convergencia de la sucesión se produce casi seguramente (es decir, que \( \lim_{n \to \infty} f_n(x) = f(x) \) para todo \( x \in \Omega \) salvo quizás en un conjunto de medida nula).

Claramente veo que la sucesión converge con la norma de \( L^p \) porque, dado que la serie es convergente, eso implica que
\( \lim_{n \to \infty} \lVert f - f_n \rVert = 0. \)
Dado que el espacio es completo sea cual sea \( p \), lo que parece es que habrá una subsucesión convergente casi seguramente y creo que el truco sería ver que la subsucesión convergente es de hecho la misma sucesión. Lo que pasa es que siento que me falta alguna hipótesis adicional (en el libro de Royden se utiliza repetidamente el concepto de "rapidly Cauchy sequence") porque no sé cómo seguir.

¿Alguna idea?

09 Septiembre, 2023, 03:45 pm
Respuesta #1

Masacroso

  • “Lo que oigo, lo olvido; lo que veo, lo recuerdo; lo que hago, lo aprendo” (antiguo proverbio chino)
  • Moderador Global
  • Mensajes: 4,607
  • País: es
  • Karma: +0/-0
Una simplificación: tomando \( g_n:=|f_n-f| \) lo que tienes que demostrar es equivalente a demostrar que si \( \sum_{n\geqslant 1}\|g_n\|_p<\infty  \) entonces \( g_n \downarrow  0 \) casi en todas partes.

A partir de ahí se puede hacer una demostración por contradicción asumiendo que el conjunto \( A:=\{x: \limsup_{n\to\infty}g_n(x)>0\} \) tiene medida positiva, entonces existe un \( \epsilon >0 \) tal que \( B:=\{x:\limsup_{n\to\infty}g_n(x)\geqslant \epsilon \} \) tiene medida positiva, de ahí se sigue que no puede darse el caso de que \( \sum_{n\geqslant 1}\|g_n\|_p<\infty  \).

09 Septiembre, 2023, 05:44 pm
Respuesta #2

girsanov

  • $$\Large \color{#5372a0}\pi\,\pi$$
  • Mensajes: 80
  • País: es
  • Karma: +0/-0
  • Sexo: Masculino
Perfecto, muchas gracias. Creo que lo he entendido, pero un par de dudillas.

1. Tomamos el límite superior de las \( g_n \) porque no sabemos si podemos asegurar que el límite tal cual existe? O hay otra razón?

2. Creo que la demostración finalizaría así, pero no estoy seguro. Lo expongo:

Tenemos que \( \mu(B)) >0 \), lo que implica que la función limitante de las \( g_n \) verificaría que no es cero en un conjunto de medida estrictamente positiva. No obstante, como la convergencia de la serie de las \( \lVert g_n \rVert_p \) implica que \( \lim_n \lVert g_n \rVert_p \), dado que la norma es un operador lipschitz podemos enchufar el límite dentro y por tanto \( \lVert \lim_n g_n \rVert_p =0. \)

Como la aplicación es una norma, \( \lVert \lim_n g_n \rVert_p = 0 \) si y solo si \( \lim_n g_n=0 \) salvo en un conjunto de medida nula. No obstante, como el conjunto \( B \) tiene medida positiva tenemos una contradicción y con eso queda finalizada la prueba.

¿Es correcto?

09 Septiembre, 2023, 07:03 pm
Respuesta #3

Masacroso

  • “Lo que oigo, lo olvido; lo que veo, lo recuerdo; lo que hago, lo aprendo” (antiguo proverbio chino)
  • Moderador Global
  • Mensajes: 4,607
  • País: es
  • Karma: +0/-0
Perfecto, muchas gracias. Creo que lo he entendido, pero un par de dudillas.

1. Tomamos el límite superior de las \( g_n \) porque no sabemos si podemos asegurar que el límite tal cual existe? O hay otra razón?

Correcto, el límite de una sucesión de números reales no tiene por qué existir, pero el límite superior (e inferior) siempre existe (aunque pueda ser infinito). Como las sucesiones en este caso son no-negativas entonces \( \lim_{n\to \infty }g_n(x)=0 \) si y solo si \( \limsup_{n\to\infty}g_n(x)=0 \), por tanto negando esto último estamos diciendo que \( g_n(x) \) no converge a cero.

Citar
2. Creo que la demostración finalizaría así, pero no estoy seguro. Lo expongo:

Tenemos que \( \mu(B)) >0 \), lo que implica que la función limitante de las \( g_n \) verificaría que no es cero en un conjunto de medida estrictamente positiva. No obstante, como la convergencia de la serie de las \( \lVert g_n \rVert_p \) implica que \( \lim_n \lVert g_n \rVert_p \), dado que la norma es un operador lipschitz podemos enchufar el límite dentro y por tanto \( \lVert \lim_n g_n \rVert_p =0. \)

No sigo muy bien tu argumento, pero no, en principio no puedes pasar el límite al interior de la norma, aunque la norma sea una función continua, porque la expresión \( \lim_{n\to \infty }g_n \) no tiene sentido.

Añadido: el tema es que la convergencia puntual no está definida en la topología dada por la norma, por tanto el único significado que se le puede dar a una expresión como \( \lim_{n\to \infty }g_n \) es el límite en norma, no algún otro tipo de límite.

Además debo añadir que la implicación \( \lim_{n\to \infty }f(x_n)=c\implies f(\lim_{n\to \infty }x_n)=c \) es falsa para funciones continuas \( f \) entre espacios topológicos ya que la sucesión \( \{x_n\}_{n\in\mathbb{N}} \) no tiene por qué poseer un límite. La implicación que sí se cumple es \( \lim_{n\to \infty }x_n=x\implies \lim_{n\to \infty }f(x_n)=f(x) \).

Citar
Como la aplicación es una norma, \( \lVert \lim_n g_n \rVert_p = 0 \) si y solo si \( \lim_n g_n=0 \) salvo en un conjunto de medida nula. No obstante, como el conjunto \( B \) tiene medida positiva tenemos una contradicción y con eso queda finalizada la prueba.

¿Es correcto?

No sería correcto por lo dicho antes. La idea de demostración anterior que escribí se puede utilizar para una demostración por contradicción o por contraposición, quizá la última forma sea más clara y directa. Es decir, si te piden demostrar que

\( \displaystyle{
\sum_{n\geqslant 1}\|g_n\|_p<0\implies g_n\to 0\text{ c.t.p. }
} \)

entonces eso es equivalente a demostrar que

\( \displaystyle{
g_n\text{ no converge a cero c.t.p. }\implies \sum_{n\geqslant 1}\|g_n\|=\infty
} \)

SOLUCIÓN BOSQUEJADA
La proposición "\( g_n \) no converge a cero casi en todas partes" es equivalente a decir que \( \mu(A)>0 \) con el conjunto \( A \) definido en mi anterior respuesta. Como \( \mu(A)>0 \) entonces (aunque no lo he demostrado habría que mostrar por qué) existe un conjunto \( B \) como el de mi anterior respuesta con medida positiva, por tanto existe una subsucesión \( \{g_{n_k}\}_{k\in\mathbb{N}} \) tal que \( \lim_{k\to \infty }\|g_{n_k}\|_p=\epsilon \), de donde se puede deducir que \( \|g_{n_k}\|_p\geqslant \|\mathbf{1}_{B}g_{n_k}\|_p\geqslant \tfrac1{2}\epsilon [\mu(B)]^{1/p} \) para \( k \) suficientemente grande (para el caso de \( p<\infty  \), y otra cosa parecida para \( p=\infty  \)) y por tanto \( \sum_{n\geqslant 1}\|g_n\|_p=\infty  \).∎
[cerrar]